Which equation has a solution of 34 for y?

Select all the correct answers.
A.8y=9
B.y−1=−14
C.4y=6
D.7−y=614
E.12y=9
F.214+y=4

Answers

Answer 1

Answer:

The answer is I don't know


Related Questions

What’s the slope intercept form?
At first I thought it was y=-4/3x+7 but it’s not. Pls help!

Answers

Answer:

y = - [tex]\frac{5}{4}[/tex] x + 7

Step-by-step explanation:

The equation of a line in slope- intercept form is

y = mx + c ( m is the slope and c the y- intercept )

Calculate m using the slope formula

m = [tex]\frac{y_{2}-y_{1} }{x_{2}-x_{1} }[/tex]

with (x₁, y₁ ) = (0, 7) and (x₂, y₂ ) = (4, 2) ← 2 points on the line

m = [tex]\frac{2-7}{4-0}[/tex] = [tex]\frac{-5}{4}[/tex] = - [tex]\frac{5}{4}[/tex]

The line crosses the y- axis at (0, 7 ) ⇒ c = 7

y = - [tex]\frac{5}{4}[/tex] x + 7 ← equation of line

HELP It's due in 15 minutes​

Answers

Answer:

L

Step-by-step explanation:

Which is the graph of the ordered pairs from the table below?

X Y
3 5
5 7
7 9
9 11

Answers

Answer:

It is the second

Step-by-step explanation:

Answer: number 2    ᕙ(@°▽°@)ᕗ

Step-by-step explanation:

((7,4),(6,3),(5,2)
Function?
Domain:
Range:

Answers

Answer:

see explanation

Step-by-step explanation:

For a function each value of x in the domain must map to exactly one unique value of y in the range.

This is the case here so it is a function.

The domain is the x- coordinates of the points

domain is { 5, 6, 7 }

The range is the y- coordinates of the points

range is { 2, 3, 4 }

A number more than 1.

Answers

Answer:

there is no more number than one

Step-by-step explanation:

Infinity and beyond!

Factorize the following: 9x²+6x+1-25y²

Answers

Answer:

(3x - 5y + 1)(3x + 5y + 1).

Step-by-step explanation:

9x²+6x+1-25y²

= 9x^2 - 25y^2 + 6x + 1

= (3x - 5y)(3x + 5y) + 6x + 1

3x + 3x = 6x and 1 * 1 = 1 so we have:

= (3x - 5y + 1)(3x + 5y + 1)

Hans Raj 4 years, 5 months ago
9x2+6x+1−25y2
.......................... (3x)2+(2x3xx1)+(1)2−25y2
.................................applying Rule (a+b)2=a2+b2+2ab
.(3x+1)2−(5y)2
........................... . (3x+1+5y)(3x+1−5y)
.................................. .... applying Rule a2−b2=(a+b)(a−b)

i need help w this pls :)

Answers

Answer:

x8 y6

Step-by-step explanation:

Please please help!!

Answers

Step-by-step explanation:

get add and add on ok

Given the function f(x)=|x|, if the function is shifted 0.5 units down, then shifted 3 units left, and reflected over the y-axis, what is the equation of the new function

Answers

Answer:

f(x) = |x - 3| - 0.5.

Step-by-step explanation:

).5 units down gives |x| - 0.5

3 units to the left gives |x + 3| - 0.5

Reflection over the y-axis gives |x - 3| - 0.5.

What type of market structure would toothpaste, jeans and laundry

detergent be apart of?

Answers

Answer:

The personal hygiene part of the store (I think)

Step-by-step explanation:

This is an ultimately weird question, so I don't really know. But my answer is what I think, (I don't even know if it exists though).

Monopolistic Competitive Market

(x+3³):(x+3)
Ehjensnsn

Answers

Answer:

x + 27

 ——————

 x + 3

Step-by-step explanation:

help please i’m doing homework and i can’t fail this lesson.

Answers

Answer: y = 3x +1

Step-by-step explanation:

2. The sum of the two number is 5 and difference of their squares is 5. Find the difference of the numbers?
help me solomon user.​

Answers

Answer:

see the attachment

I hope it's help you

Let the two numbers be X and Y

Sum of the two numbers = 5

⇛ X+Y = 5-----(1)

Difference of their squares = 5

⇛ X²-Y² = 5

⇛(X+Y)(X-Y) = 5

⇛5(X-Y) = 5

⇛X-Y = 5/5

⇛ X-Y = 1

The difference of the two numbers = 1. Ans.

also read similar questions: Two numbers differ by 5. The sum of their squares is 193. Find the numbers.

https://brainly.com/question/20616334?referrer

Lisa is using the equation x2 (x – 5)2 = 157 to find two numbers that differ by 5 and that have squares that sum to 157. when lisa solves the problem using the ...

https://brainly.com/question/1456494?referrer

I WILL GIVE BRAINLIEST Hey can someone tell me simply HOW to come to this conclusion? im a bit confiused as to how, dont want to cheat i want to learn. (P.S ive missed multiple grades which has left me with alot of gaps to fill.)

Answers

Answer:

  (b)  x +2y = 8

Step-by-step explanation:

There are at least a couple of ways to come to the conclusion.

  1) use the given information to write the equation of the desired line

  2) check to see which of the answer choices satisfies the requirement

__

1)

The given line has two specified points on it. These can be used to find its slope. (Or, you can count grid square to see it has a rise of -1 for a run of +2.)

The slope formula is ...

  m = (y2 -y1)/(x2 -x1)

  m = (-4 -0)/(4 -(-4)) = -4/8 = -1/2

Comparing this to our grid square count, we see they are the same:

  m = rise/run = -1/2

Now, we have a slope (-1/2) and a point (2, 3), so we can use these to write the point-slope equation of the desired line:

  y -k = m(x -h) . . . . . . . line with slope m through point (h, k)

  y -3 = -1/2(x -2) . . . . . point-slope equation of the desired line

  y -3 = -1/2x +1 . . . . . . eliminate parentheses

  1/2x +y = 4 . . . . . . . . add 1/2x +3 to both sides

  x +2y = 8 . . . . . . . . . multiply by 2

__

2)

Perhaps easier in this case is to try the given equations to see which is satisfied at the given point. There are two different left-side expressions to evaluate at the point (x, y) = (2, 3):

  x +2y = (2 +2(3)) = 2+6 = 8

  2x +y = 2(2) +3 = 4 +3 = 7

Only the equation ...

  x +2y = 8

is satisfied at the given point.

_____

Additional comments

While I encourage you to learn to solve questions like this "from scratch", modern math curriculum is often "multiple choice."

When faced with multiple choices for answers, it is often easiest to ...

  (a) eliminate the ones that are not self-consistent or that are false on their face;

  (b) eliminate the ones that have an obviously fatal flaw (wrong sign of slope, too large or too small, for example);

  (c) choose from the remaining ones the one that properly answers the question (satisfies all requirements).

__

When you solve a standard-form equation for y, you get ...

  ax +by = c

  by = -ax +c

  y = -a/bx +c/b

That is, the slope of the line is -a/b.

For the two left-side expressions in the answer choices, the slopes are ...

  x +2y   ⇒   slope = -1/2

  2x +y   ⇒   slope = -2/1 = -2

The scales on the graph are the same in the x- and y-directions, so the shallow negative slope of the given line is clearly less than 1. That eliminates the last two answer choices already.

Will mark brainliest if you can resolve this problem ( 2 possible answers, NO LINKS OR ANSWERS WITHOUT EXPLANATION)
I can give u the permission of drawing the boxplot by:
- saving the picture or screenshot it ( it's better to screenshot it (Windows+ Logo+Shift+s) )
- drawing the answer on the screenshot Windows
- save the picture (Ctrl+s)
- and finally send it to brainly
(P.s: many people (those who are good at I.T) will argue with the procedure of doing the answer. But many people don't. So I gave the procedure step-by-step to you to show me the working) Thx
Here's the picture:

Answers

Answer:

Hopefully this is what you are looking for.

Step-by-step explanation:

Hope this helps!

Find two numbers that round to 387.4 when rounded to the nearest tenth. Drag the numbers to the box.

Answers

Answer: By using the rounding rule.

Step-by-step explanation:

387 x 397 => 387.4

384 x 352 => 387.4

Evaluate this -4-12/(-4)?

Answers

Answer:

4

Step-by-step explanation:

how
5. Shane has no more than $200 to spend on a
basketball game for next weekend. He spends
$44 on fees. If the tickets are sold at $78 each,
which inequality models t, the number of tickets
Shane is able to buy?

Answers

Answer:

[tex]t\leq2[/tex]

Step-by-step explanation:

200 - 44 = 156 dollars which Shane can use to buy tickets

156 >= 78t

t<=2

Therefore the maximum number of tickets he can buy is 2

how do i tell if an angel is comolentery, supplmentsry, or neither? I was not at school the day they showed me this? ​

Answers

Answer:

supplementary angles - two angles are supplementary when they add up to 180 degrees (a straight line)

complementary angles - two angles are complementary when they add up to 90 degrees (a right angle)

hopefully this helps! :)

Answer:

1. complementary

2. neither

3. supplementary

4. complementary (x = 6)

10x + 30 = 90         x + 3 = 9          x = 6

5. supplementary (x = 20)

4x + 40 + 3x = 180       7x = 140       x = 20

6. x = 50.5

2x + 79 = 180     2x = 101       x = 50.5

Step-by-step explanation:

When the measurement of the angles adds up to 90 degrees, the angles are complementary. When the sum of the angles is 180 degrees, then they are supplementary.

On September 1, you own 12.5 shares of stock that are worth $149.87. On
December 1, the shares are worth $94.87. What was the change in the price per
share of stock?

Answers

Divide $149.87 by 3 which = $49.95
From September 1 to December 1 is three months apart so meaning the price dropped $49.95 for the months of Oct to Dec.

The required change in the price of the share of stock is $4.4.

Given that,
On September 1, you own 12.5 shares of stock that are worth $149.87. On December 1, the shares are worth $94.87, and the change in the price per share of stock is to be determined.

What is simplification?

The process in mathematics to operate and interpret the function to make the function or expression simple or more understandable is called simplifying and the process is called simplification.

Here,
Initial total cost shares of stock = $149.87
A total number of shares = 12.5
The initial cost of single share = 149.87/ 12.5 = $11.98
Now final cost of the total share = $94.87
The final cost of the single share is = 94.87 / 12.5 = $7.58
Change in price of shares of stock = 11.98 - 7.58 = $4.4

Thus, the required change in the price of the share of stock is $4.4.

Learn more about simplification here: https://brainly.com/question/12501526

#SPJ6

As a car owner/lessee, it is not a requirement to have minimum coverage auto insurance.

T or F

Answers

Answer: False

Step-by-step explanation:

empat angka setelah angka tiga

Answers

Answer:

hindi ko po gets

yan sorry

10/11 x 3/7 x 1/2 Multiply. Write your answer as a fraction in the simplest form.

Answers

The simplest form to this equation would be 15/77

please mark me brianliest if i was correct!

Fractions are written as a ratio of two integers. The product of the fractions in its simplest form is 15/77

Simplification of fractions

Fractions are written as a ratio of two integers

Give the expression

10/11 x 3/7 x 1/2

This can also be written as:

10/2 * 1/11 * 3/7

= 5/11 * 3/7

= 15/77

Hence the product of the fractions in its simplest form is 15/77

Learn more on fractions here: https://brainly.com/question/78672

#SPJ2

Which of the following functions is graphed below

Answers

Answer:

Option B: [tex]y=\left \{ {{x^2+6,x<3} \atop {-x+6,x\geq3}} \right.[/tex]

Step-by-step explanation:

We can see that the first part of the piecewise function is a parabola with a y-intercept of (0,6) where the domain is restricted for all values of x less than 3 given the open circle at x=3.

The second part of the piecewise function is a linear function with a negative slope where the domain is restricted for all values of x greater than or equal to 3 given the closed circle at x=3.

Essentially, the piecewise function has a jump discontinuity at x=3 where the first part of the function goes to the second part of the function at one point by "jumping".

30% of 70 is what?
me is not smart

Answers

Answer:

21

Step-by-step explanation:

30/100 × 70= 2100/100

= 21

For each problem below put the lettered measures in order for greatest to the least

Answers

Angles can be used to estimate the side lengths of a shape.

The letters in order of greatest to least are (a), (b) and (c)

The sum of angles at side (a) is:

[tex]a = 30 + 50[/tex]

[tex]a = 80[/tex]

The sum of angles at side (c) is:

[tex]c = 50 + 90[/tex]

[tex]c = 140[/tex]

The sum of angles at side (b) is:

[tex]b = 40 + 90[/tex]

[tex]b = 130[/tex]

So, we have:

[tex]a = 80[/tex]

[tex]b = 130[/tex]

[tex]c = 140[/tex]

The smaller the base angles, the longer the side.

Hence, the letters in order of greatest to least are (a), (b) and (c)

Read more about angles at:

https://brainly.com/question/11585894

What is the volume of this container?

Answers

the answer is 1,875 cm^3 because if you multiply the length of each side of the large cube and subtract the volume of the smaller cube, you’ll get the total volume
1,875 is the answer

Use the linear combination method to solve the system of equations. â€"2. 1x 4y = 5. 3 2. 1x â€" 5. 5y = â€"0. 2 What is the solution to the system? ( , ).

Answers

The solution to the systems of equations is (7, 3)

Given the systems of equations expressed as:

-x + 4y = 5 ....................1

x - 5y = -2 ...................... 2

Add both equations to have:

-x+x + 4y - 5y = 5 - 2

4y-5y = -3

-y = -3

y = 3

Substitute y = 3 into equation 1:

-x + 4(3) = 5

-x + 12 = 5

-x = 5 - 12

-x = -7

x = 7

Hence the solution to the systems of equations is (7, 3)

Learn more on simultaneous equations here: https://brainly.com/question/148035

What is an equation of the line that passes through the points (0, -4) and (-4, 6)?

Answers

I got this answer by first using the slope formula, which would result into [tex] \frac{0 - 4}{6 - 0} \\ [/tex]for which the answer would be - ⅔ Then, the y-intercept could be found easily, since you already have it., which is (0,4). Since the format for all linear equations are y=mx+b , in which b means the y-intercept and m means the slope. So then, if you substitute - ⅔ for m and 4 for b , you would get

[tex]y = - \frac{2}{3} x + 4 \\ [/tex]

Solve the system of linear equations by substitution
6x-9=y
y=-3x

Answers

Answer:USE SUBSTITUTION METHOD. Algebra work right here

Step-by-step explanation:

Other Questions
What was a result of the efforts of john steinbeck, dorothea lange, and woody guthrie?. A tank contained 35 liters of water. After some water was poured out, it was 2/5 filled. How much water was poured out from the tank? help me please asapi will make you brainliest Choose two spheres of Earth which interact when hot springs are formed. Use complete sentences to explain how they interact during this process. Please help me solve this Why is it important for all the phases of growth and reproduction to proceed correctly in order for a plant species to survive? For any two numbers a and b, the product of a b times itself is equal to a^2 2ab + b^2. Does this familiar algebraic result hold for dot products of a vector u v with itself? In other words, is it true that (u v) (u v) = u u 2u v + v v? Justify your conclusion, trying not to express vectors u and v in component form. 50 PIONTS PLZ HELPPretend you are the chairperson for your community group and your organization has decided to plan an event to celebrate your town/city. Consider the reason for your community event (Homecoming, Stanley Cup Parade from a local NHL star, Grand Opening of a new facility, Holiday, etc). *If you would like to come up with your own meeting topic Email Me or Book a Talk*Your role is to create the agenda for the first meeting using the template provided in the following section (this is to be handed in).Choose one area of the agenda that you will orally present (this will probably be the introduction or the closing portion of the meeting- but it could be the trouble-shooting portion if you are more drama minded). This weekend, Jerome earned $252.50 in all. What was the dollar amount of the meals that Jerome served? How is computer science helping this scientist do her research? Write a system of equations to describe the situation below, solve using elimination, and fill inthe blanks.The manager at a community pool is looking over receipts. On a certain Monday, the pool had16 children and 42 adults, which brought in $142. That same week on Tuesday, 24 childrenand 26 adults came to the pool, which brought in $102. What are the admission prices forchildren and adults? Kenya uses mental math to find the sum of 235 and 23. She uses 235,20 and one other number. What is the missing number Max worked for 2/3 hour. It takes 1/6 hour to make one card. How many cards has Max made? Provide students with fraction bars or guide them in creating a tape diagram as they work in pairs to solve. how do you evalute the political development of our country Can someone help me solve this problem? Im not sure how to solve this type of expression the first international passenger air service traveled along which route? The story has an ambiguous ending that leaves the outcome up to the readers imagination. Based on your understanding of the characters, as well as your knowledge of plot and setting, write a continuation of the story that tells what you think will happen after Okekes revelation. 21 cans of tomato paste of the same size have weight of 7300 grams. What is the weight of 5 cans. Which of the following is true of (1, 4) and (1, 7)?. what is the relation between population and economic?